What is 60,000 + 3,000 + 20 + 9 in standard form

Answers

Answer 1

Answer:

the standard form is 63029

Answer 2
63029 is the answer in standard form

Related Questions

the width w of a rectangular swimming pool is x+5 the area a of the pool is x^2+4-5

what is an expression for the length of the pool? step by step

Answers

Answer:

x-1

Step-by-step explanation:

Area= x^2+4x-5

Width= x+5

Area= Length*breadth

x^2+4x-5=length*x+5

Length=x^2+4x-5/x+5

Therefore, length= x-1

Answer:

x-1

Step-by-step explanation:

i got this problem on khan academy

Question on file. Thx in advance

Answers

The answer is 1,320
Expiation: 22x60= 1,320
Since there are 60 min in a hour you must multiply the 22L produced in a minute by 60
The answer is 1,320

The area of the base of a right square pyramid is multiplied by 5, but the height is unchanged. What is the volume of the new pyramid divided by the volume of the original pyramid?

Answers

9514 1404 393

Answer:

  5

Step-by-step explanation:

The volume of a right square pyramid is given by the formula ...

  V = 1/3Bh

where B is the area of the base.

The volume of the larger pyramid will be ...

  V' = 1/3(5B)h

So, the ratio of volumes is ...

  V'/V = (1/3(5B)h)/(1/3Bh)

  V'/V = 5

The ratio of volumes is 5.

The ratio of volumes is 5

Rule 1: Multiply by 2, then add one third starting from 1. Rule 2: Add one half, then multiply by 4 starting from 0. What is the fourth ordered pair using the two sequences? (5, 10) (four and two thirds, 42) (21, 170) (two and one third, 2)

Answers

2 is the correct answer 2 is the correct answer
the answer your looking for is 2

please help -

Mr. Day bought a lawn mower with his credit card for $400. The interest rate on the card is 24% (0.24) per year. If payment is not made on time, the credit card company charges a late fee of $30. Mr. Day does not make a payment for the first month. What would be the interest, late fee, and new total he owes?

Answers

Answer: ok fine

Step-by-step explanation:

can someone help please (highlighted in blue)

Answers

[tex]\implies {\blue {\boxed {\boxed {\purple {\sf {B)\:10}}}}}}[/tex]

[tex]\large\mathfrak{{\pmb{\underline{\red{Step-by-step\:explanation}}{\red{:}}}}}[/tex]

[tex]4 + 3 \: ( \: 10 - {2}^{3} \: )[/tex]

➺[tex] \: 4 + 3 \: ( \: 10 - 2 \times 2 \times 2 \: )[/tex]

➺[tex] \: 4 + 3 \: ( \: 10 - 8 \: )[/tex]

➺[tex] \: 4 + 3 \: ( \: 2 \: )[/tex]

➺[tex] \: 4 + 6[/tex]

➺[tex] \: 10[/tex]

Note:-

[tex]\sf\pink{PEMDAS\: rule.}[/tex]

P = Parentheses

E = Exponents

M = Multiplication

D = Division

A = Addition

S = Subtraction

[tex]\large\mathfrak{{\pmb{\underline{\orange{Mystique35 }}{\orange{❦}}}}}[/tex]

Answer:

10

Step-by-step explanation:

Use

Parenthesis

Exponents

Multiplication(Left - Right)

Division(Left - Right)

Addition(Left - Right)

Subtraction(Left - Right)

A.1.5
B.9
C.20
D.22
plsss answer as fast as u can

the first person that answers and there answer is correct i will give u brainliest

Answers

Answer:

x=50 y=10

Step-by-step explanation:

the answer is D, because when you look at the chart






show all your work please and thank you let me show you an example of showing your work

the second one is an example from my other one I just asked

Answers

Answer:

so we jsut show the work for the first question I'll write in the comment box then

Answer:

700 cubic inches

Step-by-step explanation:

The question asks for how much volume the box can hold. We multiply the length (14), width (10), and height (5) to get the volume. 14 · 10 · 5 = 700.

The box can hold 700 cubic inches.

The table shows the number of practice problems completed in 30 minutes in three samples of 10 randomly selected math students.
Which statement is most accurate based on the data?

Answers

Answer: A

Step-by-step explanation: As it is a data set that has been equally represented, it is ok to predict.

Answer A step by step explanation as it is set that has been equaling resendted it is ok to predict

(Geometry Question) : Given the points A(-3, -4) and B(2, 0), find the coordinates of the point P on directed line segment AB that partitions AB in the ratio 2:3.

Answers

Answer-7:4


Have a good day
7:4
Mark me Branalist pls

The prices for 7 different totes are $52,$22, $50, $44, $60, $48, and $52. What is the outlier in the data set?

Answers

Answer:

$22

Step-by-step explanation:

An outlier in a data set is the one farthest from the other data points. 22 is far away from the other points, so that is the outlier.

Answer:

22

Step-by-step explanation:

It is farther off than from all of the rest of the #'s.

how do I graph k(x)=13 x


show work plz.

Answers

Answer:

Step-by-step explanation:

Use the slope and y intercept, or 2 points

Slope: 13

y-int: 0,0

x|y

0|0

1|13

ANSWER- 0|0

Explainion - :))

One staple weighs 31 milligrams. If a box of staples holds 250 staples, how many grams of staples does the box hold?
A: 6.75
B: 7.75
C: 67.5
D: 77.5
E: 7,750
If you get this right i will give you the brainliest!

Answers

Answer:

7.75

Step-by-step explanation:

1 mg = 0.001 g

31 mg * 0.001 = 0.31 grams per staple

0.31 g * 250 = 7.75 grams per 250 staples

Answer:

b) 7.75

Step-by-step explanation:

convert 31 milligrams to grams = .031 grams

250 x .031 = 7.75 grams

If u={a,b,c,d}and p={b,d,e}find p(bar at the top)

Answers

Answer:

{a , c}

Step-by-step explanation:

first find u intersection p

n(U n P ) = {a , b , c , d} n {b , d , e}

={d , b}

intersection means element or values which lies in both the sets

now to find p complement

note : p( bar at the top ) is read as p complement

p complement = n(U) - n(U n P)

={a , b , c , d} - {d , b}

={a , c}

n(U) -n(U n P) means the elements which is only in n(U ) but not in n(U n P)

Answer:

a,c

Step-by-step explanation:

GIVING OUT BRAINLIEST / 60 POINTS TO WHOEVER ANSWERS

Answers

Answer:

6 2/3 = x

Step-by-step explanation:

24 1/14 + 8 3/7  = (x ÷ 1 1/9) * 5 5/12

Add on the left by getting a common denominator

24 1/14 + 8 3/7*2/2  = (x ÷ 1 1/9) * 5 5/12

24 1/14 + 8 6/14  = (x ÷ 1 1/9) * 5 5/12

32 7/14  = (x ÷ 1 1/9) * 5 5/12

32 1/2 =  (x ÷ 1 1/9) * 5 5/12

Changing to improper fractions

(2*32+1)/2 = (x ÷ 10/9) * (12*5+12)/12

65/2 = (x ÷ 10/9) * (65)/12

Multiply each side by 12/65

65/2 * 12/65 = (x ÷ 10/9) * (65)/12*12/65

6 = (x ÷ 10/9)

Copy dot flip

6 = x * 9/10

Multiply by 10 /9

6 * 10/9 = x * 9/10 * 10/9

60/9 =x

20/3 =x

Changing back to a mixed number

3 goes into 20 6 times with 2 left over

6 2/3 = x

Answer:

[tex]\frac{20}{3}[/tex]

Step-by-step explanation:

[tex](24+8)+(\frac{1}{14} +\frac{3}{7})=(x:\frac{10}{9}).\frac{65}{12}\\32+\frac{1}{2}=(x.\frac{9}{10}) .\frac{65}{12} \\32\frac{1}{2}=(\frac{9}{10}x).\frac{65}{12} \\\frac{65}{2}=\frac{9}{10}x.\frac{65}{12} \\\frac{65}{2}=\frac{3}{2} x.\frac{13}{4} \\\frac{65}{2} =\frac{39}{8} \\\frac{20}{3} =x\\x=\frac{20}{3}[/tex]

el   . es multiplicación

Write the number in 2 equivalent forms as a​ fraction, decimal, or percent.
11​%
What is the equivalent​ decimal?

Answers

Answer:

[tex] \frac{11}{100} \: \: or \: \: 0.11[/tex]

Step-by-step explanation:

11%

=11/100

=0.11

.11 or 11/100 because a percentage is from a hundred.

Members of a lacrosse team raised $1352.50 to go to a tournament. They rented a bus for $802.50 and budgeted $25 per player for meals. Write and solve an equation which can be used to determine pp, the number of players the team can bring to the tournament.

Answers

Answer:

f(25)1352.50+801.50

=25x+2154

the answers is in the photo and it’s done by muhammad

HELP PLEASEEEEEEEEEEEEEEEEEEEEE

Answers

Answer:

well the height of the triangle would be 5, the base is 12. The area of a triangle is the height times the base divided by 2 so 12 times 5 = 60 divided by 2= 30.

Answer:

A=30

Explanation:

b Base

12

hb Height

5

Solution

A=hbb

2=5·12

2=30

I think is like this. >3

plz correct ans only thx for the help

Answers

Answer:

The width is 5m (A)

Step-by-step explanation:

The formula for area is length times width times height.

LxWxH

I hope this helps!

The area of a floor is 49m2. If the length is 8m, what is the width?
ANSWER: A.) 40 ÷ 8 = 5 m
Explanation:
A floor is a rectangle
The area of a rectangle is length • width
So to find the width, divide the area by the length:
40 ÷ 8= 5 m

A container holds 1 3/4 of litres of water. How many containers can be filled from 49 litres of water?
ANSWER: 28 containers
Explanation: the question is asking how many 1 3/4 liters of water-filled containers will be needed to have a total of 49 litres of water.

49 litres ÷ 1 3/4 litres = 28

surds.... plzz help, matching up, thank you

Answers

1 one to D
2 one to B
3 one to C
4 one to A

Answer:

1 - D

2 - B

3 - C

4 - A

I hope this helped!

the length of a rope is 5y meters.
from one end of the rope, 2y meters have been cut and from the other end x meters have been cut will mark brainlest

Answers

The answer is x3 in meters
Since the rope is 5y meters in total,
and 2y meters was taken off one side,
5 - 2 = x

The other end or the rope that’s left or cut would be 3y meters

So the answer:
x: 3y meters

Which sentence correctly compares the two points shown on the number line? -1 > 0.25 , -1 < -0.25, -0.25 < -1

Answers

Answer:

-1 < -0.25

Step-by-step explanation:

-1 is less than -0.25

less than symbol = <

greater than symbol = >

Example; -1 > -2 --- negative one is greater than negative two OR -2 < -1 --- negative two is less than negative one

Answer:

-1<-0.25

Step-by-step explanation:

Find the dfference between (8a + 2b - 4) and (3b - 5)

Answers

Answer:

8a-b+1

Step-by-step explanation:

Difference means subtract

(8a + 2b - 4) - (3b - 5)

Distribute the minus sign

(8a + 2b - 4) - 3b + 5

Combine like terms

8a + 2b-3b - 4  + 5

8a-b+1

hi i’m so sorry for this. i just need points so i can message someone

Harry collects data from a random sample of seventh-graders. Out of 25 respondents, 10 ride the bus home. Of the 150 seventh-graders who attend Harry’s school, how many would be expected to ride the bus to school?

Answers

You would divide 150 by 25 which equals 6 and times it by 10 so it would be 60

Select the correct answer.
Which table represents the increasing linear function with the greatest unit rate?
A.
x y
2 16
5 10
B.
x y
2 14
6 12
C.
x y
2 -24
5 -15
D.
x y
2 -12
6 -16
E.
x y
2 -19
6 -17

Answers

Answer:

E

Step-by-step explanation:

Answer:

Option c is the correct answer if the answer is correct plz mark me as brainliest.

There are 8 teachers in a school, 4 males and 4 females. 4 teachers are chosen to join a committee. How many ways are there when 2 males and 2 females chosen?

Answers

It is 4 dude this simple math it is basicly subtraction anyway pls give me brainlest

Answer:

Step-by-step explanation:

8 TEACHERS

4 MALES 4 FEMALES

4 ARE GONE SO 2 MALES LEFT 2 FEMALES LEFT =4 TEACHERS

What is the volume of the composite figure shown? Use 3.14 for π.

Answers

Answer:

25.12

Step-by-step explanation:

the formula of a cone is pi r^2 * h/3

r = radius and h = height

the radius is 2 and the height is 3

and the question says use 3.14 instead of pi so

we have

3.14 * 2^2 * 6/3

2*2 or 2^2 = 4

3.14 * 4 * 6/3

6/3 = 2

3.14 * 4 * 2

4*2 = 8

and 8 * 3.14 is 25.12

The answer to this question is 25.12

How many times as great as
2 x 103 is 8 x 105?

Answers

2 x 103 = 206, 8 x 105 is 840.
840 is roughly 4.08 times greater than 206.

Answer:gấp 103/420

Step-by-step explanation:2*103 =206

8*105=840 rồi lấy 206/840 =103/420 đáp án

can yall please help me, include steps

Answers

It’s A because it’s A have a good day
answer is A because they make a straight line

SOLVE AND NO LINKS
x – 3 = 5

Answers

Answer:

8 = x

or

x = 8

Step-by-step explanation:

x – 3 = 5

x + -3 + 3 = 5 + 3

x = 5 + 3

(5 + 3 = 8)

x = 8

Answer:

x=8

Step-by-step explanation:

x-3=5

x-3+3=5+3

x=8

Other Questions
RST Company produces a product that has a variable cost of $6 per unit. The company's fixed costs are $30,000. The product sells for $10 per unit. The break-even point in sales dollars is $_____________. Please help me solve this. I keep getting the answer weong What is f(-2) for f(x)=(1/2)x^2 What is the explicit formula for this arithmetic sequence?-10,-17, -24, -31 binomial theorem to expand (x+4)^4 Help pls!! this is a presentation Im doing in class and I NEED this completed rn, ASAP!!! . What I need is just the answers for the picture below and I NEED the explanation to all of them, and if you can pls try to give a similar example to these and solve it or what did u use to solve it. (Only 1 example that is similar) Thank you so much! You have a (12x+8) and 55* degree and (8x-3) in a equal triangle right like this /\ and have to solve for xplease help The earliest newsletters were inscribed by educated slaves.FACTOPINION Count the number of breaths you take in a minute and compare it with the number of times a fish opens and closes its mouth On January 1, 2019, Sunland Company granted Sam Wine, an employee, an option to buy 1,000 shares of Sunland Co. stock for $30 per share, the option exercisable for 5 years from date of grant. Using a fair value option pricing model, total compensation expense is determined to be $5520. Wine exercised his option on October 1, 2021 and sold his 1,000 shares on December 1, 2021. Quoted market prices of Sunland Co. stock in 2021 were: One number is 8 times a first number. A third number is 100 more than the first number. If the sum of the three numbers is 1090, find the numbers. Cunto mide el lado de un terreno cuadrado, si su rea 144 m2? Consider angle in Quadrant II, where cos=12/13. What is the value of sin? Peter Piper picked a pickled pepper out of a pepper jar. If the probability of drawing a pickled pepper was 2/5,how many total peppers could be in the jar (psst. you can't have a half of a pepper)? Select the correct answer. If pinecones are placed in water, they close up. But they open up in warm and dry conditions. How do pinecones benefit from this characteristic? choices in pic Based on the picture below (note that it is not drawn to scale), Maggie says that . Is she correct? Explain why or why not. Anna's electricity bill costs $21.59 per month plus $4.66 per kilowatt hour. How many kilowatt hours can she use and keepher bill to no more than $51?Round your answer to a whole number Consider the trapezoid shown below.6 in.4 in.6 in.6 in.(not drawn to scale) 2. Do you agree with the results of Schenck's case? Why or why not?help plzzz Which of these is a feature of Gothic writing that makes it different from otherRomantic genres?